Integration Problem: Solving ∫ 1/r³ dr/dt

  • Thread starter Thread starter kashiark
  • Start date Start date
  • Tags Tags
    Integration
Click For Summary
SUMMARY

The discussion centers on solving the integral ∫ 1/r³ dr/dt, where the lack of a direct equation for r in terms of t complicates the integration. Participants clarify that the integral can be rewritten as ∫ dt (1/r³ dr/dt), allowing the use of the Fundamental Theorem of Calculus (FTC) to find the anti-derivative. The solution is expressed as -1/(2r²) + C, demonstrating a valid approach despite initial confusion regarding notation and integration techniques.

PREREQUISITES
  • Understanding of calculus, specifically integration techniques.
  • Familiarity with the Fundamental Theorem of Calculus (FTC).
  • Knowledge of chain rule differentiation.
  • Basic concepts of physical situations involving variable rates of change.
NEXT STEPS
  • Study the Fundamental Theorem of Calculus in detail.
  • Learn about chain rule differentiation and its applications in integration.
  • Explore variable substitution techniques in integrals.
  • Investigate physical applications of integrals in motion and rates of change.
USEFUL FOR

Students studying calculus, particularly those tackling integration problems involving variable rates, as well as educators seeking to clarify integration techniques and the application of the Fundamental Theorem of Calculus.

kashiark
Messages
210
Reaction score
0

Homework Statement


∫ 1/r³ dr/dt


Homework Equations



∫ xa dx = x(a+1)/(a+1)

The Attempt at a Solution


I have no clue how to solve it like this. I don't have the equation for r in terms of t, so I can't just substitute. How would I do it?
 
Physics news on Phys.org
kashiark said:

Homework Statement


∫ 1/r³ dr/dt


Homework Equations



∫ xa dx = x(a+1)/(a+1)

The Attempt at a Solution


I have no clue how to solve it like this. I don't have the equation for r in terms of t, so I can't just substitute. How would I do it?

is that how the question is actually written & what are you integrating with respect to?

it doesn't really make sense how you have written it, do you mean:
\int dt (\frac{1}{r(t)^3}\frac{dr}{dt})

if so have a think about how chain rule differentiation works & how you could try & reverse it using FTC...
 
OK if it were like that, wouldn't the dt's just divide out allowing you to integrate with respect to r?
∫ dt(1/r³ dr/dt)
∫ 1/r³ dr
-1/(2r²) + C
 
Last edited:
you can, but its a bit of an abuse of notation, athough it gives the same answer, a better way to think of it is to write:

\frac{d}{dt} (-\frac{1}{2r(t)^2}) = \frac{1}{r(t)^3}\frac{dr}{dt}

then the intergal becomes
\int dt (\frac{d}{dt} (-\frac{1}{2r(t)^2}))

so by FTC, the anti-derivative is
= (-\frac{1}{2r(t)^2}) +C

though the question was to you - how is it written in the actual problem?
 
It's not on a worksheet or anything; I was just messing with a particular physical situation, and I came to something in that form that I needed to integrate.
 
Question: A clock's minute hand has length 4 and its hour hand has length 3. What is the distance between the tips at the moment when it is increasing most rapidly?(Putnam Exam Question) Answer: Making assumption that both the hands moves at constant angular velocities, the answer is ## \sqrt{7} .## But don't you think this assumption is somewhat doubtful and wrong?

Similar threads

  • · Replies 5 ·
Replies
5
Views
2K
Replies
9
Views
2K
  • · Replies 9 ·
Replies
9
Views
2K
  • · Replies 10 ·
Replies
10
Views
1K
Replies
2
Views
1K
  • · Replies 105 ·
4
Replies
105
Views
6K
Replies
0
Views
1K
  • · Replies 14 ·
Replies
14
Views
2K
  • · Replies 3 ·
Replies
3
Views
2K
  • · Replies 12 ·
Replies
12
Views
2K